Last visit was: 03 Jun 2024, 13:27 It is currently 03 Jun 2024, 13:27
Close
GMAT Club Daily Prep
Thank you for using the timer - this advanced tool can estimate your performance and suggest more practice questions. We have subscribed you to Daily Prep Questions via email.

Customized
for You

we will pick new questions that match your level based on your Timer History

Track
Your Progress

every week, we’ll send you an estimated GMAT score based on your performance

Practice
Pays

we will pick new questions that match your level based on your Timer History
Not interested in getting valuable practice questions and articles delivered to your email? No problem, unsubscribe here.
Close
Request Expert Reply
Confirm Cancel
SORT BY:
Date
Intern
Intern
Joined: 26 Feb 2017
Posts: 24
Own Kudos [?]: 201 [68]
Given Kudos: 43
Location: Brazil
GMAT 1: 610 Q45 V28
GPA: 3.11
Send PM
Most Helpful Reply
Manager
Manager
Joined: 15 Feb 2017
Posts: 57
Own Kudos [?]: 405 [35]
Given Kudos: 35
Send PM
Math Expert
Joined: 02 Sep 2009
Posts: 93473
Own Kudos [?]: 628902 [12]
Given Kudos: 82085
Send PM
General Discussion
Intern
Intern
Joined: 26 Feb 2017
Posts: 24
Own Kudos [?]: 201 [2]
Given Kudos: 43
Location: Brazil
GMAT 1: 610 Q45 V28
GPA: 3.11
Send PM
Re: If |x–1|<4, so: [#permalink]
2
Kudos
Bunuel wrote:
vitorpteixeira wrote:
If |x–1|<4, so:

(A) x > 3
(B) x ≤ 4
(C) -4< x < 4
(D) x > -4
(E) none of the above


\(|x–1|<4\);

\(-4<x-1<4\);

\(-3<x<5\).

Any x from that range will be more than -4.

Answer: D.

P.S. What is the source of this question?





Bunnel,

First of all, thank you very much.

The source of this questions is a bank of questions that I received from a friend, whose his teacher gave to him.

I have huge doubts on inequalities topics.

As you explained above any value X>-4 can satisfy this.

What is the best approach to deal with this kind of questions? I solved -3<x<5, but I could not figure out, X>-4 because -3>-4 and do not satisfy the inequality.

Can you please help?

I have the same doubt on this question, below...

https://gmatclub.com/forum/if-z-2-4z-5- ... l#p1949237
Math Expert
Joined: 02 Sep 2009
Posts: 93473
Own Kudos [?]: 628902 [4]
Given Kudos: 82085
Send PM
If |x–1|<4, so: [#permalink]
1
Kudos
3
Bookmarks
Expert Reply
vitorpteixeira wrote:
Bunuel wrote:
vitorpteixeira wrote:
If |x–1|<4, so:

(A) x > 3
(B) x ≤ 4
(C) -4< x < 4
(D) x > -4
(E) none of the above


\(|x–1|<4\);

\(-4<x-1<4\);

\(-3<x<5\).

Any x from that range will be more than -4.

Answer: D.

P.S. What is the source of this question?





Bunnel,

First of all, thank you very much.

The source of this questions is a bank of questions that I received from a friend, whose his teacher gave to him.

I have huge doubts on inequalities topics.

As you explained above any value X>-4 can satisfy this.

What is the best approach to deal with this kind of questions? I solved -3<x<5, but I could not figure out, X>-4 because -3>-4 and do not satisfy the inequality.

Can you please help?

I have the same doubt on this question, below...

https://gmatclub.com/forum/if-z-2-4z-5- ... l#p1949237


It's the other way around. We know that \(-3<x<5\). Any x from that range satisfy x > -4.


Check similar questions:
https://gmatclub.com/forum/if-it-is-tru ... 81702.html
https://gmatclub.com/forum/if-xy-0-and- ... 39194.html
https://gmatclub.com/forum/if-it-is-true ... 29093.html
https://gmatclub.com/forum/if-it-is-tru ... 66196.html
https://gmatclub.com/forum/if-4x-12-x-9- ... 01732.html
https://gmatclub.com/forum/if-4-7-x-3-w ... 68681.html
https://gmatclub.com/forum/if-5-x-0-whi ... 31556.html
https://gmatclub.com/forum/if-it-is-tru ... -9602.html
https://gmatclub.com/forum/if-m-0-which ... 54150.html
https://gmatclub.com/forum/if-x-0-which ... 76331.html
https://gmatclub.com/forum/if-x-3-1-2-w ... 29673.html
Manager
Manager
Joined: 02 Jul 2016
Posts: 167
Own Kudos [?]: 59 [0]
Given Kudos: 67
Location: India
GMAT 1: 650 Q49 V28
GPA: 4
Send PM
Re: If |x–1|<4, so: [#permalink]
Bunuel wrote:
If |x–1|<4, so:

(A) x > 3
(B) x ≤ 4
(C) -4< x < 4
(D) x > -4
(E) none of the above


\(|x–1|<4\);

\(-4<x-1<4\);

\(-3<x<5\).

Any x from that range will be more than -4.

Answer: D.

P.S. What is the source of this question?[/quote]



Hi, Bunuel
Just wanted to ask that since in this question it is not mentioned that x is an integer, by the range x>-4 it will include values greater than -3 also (say -3.8) but our range is from -3 to 5 so isnt this wrong????
Math Expert
Joined: 02 Sep 2009
Posts: 93473
Own Kudos [?]: 628902 [0]
Given Kudos: 82085
Send PM
Re: If |x–1|<4, so: [#permalink]
Expert Reply
Target Test Prep Representative
Joined: 04 Mar 2011
Status:Head GMAT Instructor
Affiliations: Target Test Prep
Posts: 3042
Own Kudos [?]: 6418 [4]
Given Kudos: 1646
Send PM
Re: If |x–1|<4, so: [#permalink]
3
Kudos
1
Bookmarks
Expert Reply
vitorpteixeira wrote:
If |x–1|<4, so:

(A) x > 3
(B) x ≤ 4
(C) -4< x < 4
(D) x > -4
(E) none of the above


We can solve the inequality when (x - 1) is positive and when it is negative.

When (x - 1) is positive:

x - 1 < 4

x < 5

When (x - 1) is negative:

-x + 1 < 4

-x < 3

x > -3

-3 < x < 5

Answer: D
Manager
Manager
Joined: 05 Dec 2016
Posts: 194
Own Kudos [?]: 89 [1]
Given Kudos: 49
Concentration: Strategy, Finance
GMAT 1: 620 Q46 V29
Send PM
Re: If |x–1|<4, so: [#permalink]
It's a tricky question, I think I've encountered a sort of this one in OG2017.
Key to solve it is to find the range from the answer choices that covers all possible values of x,here it is answer choice D.
Intern
Intern
Joined: 24 Nov 2016
Posts: 29
Own Kudos [?]: 11 [2]
Given Kudos: 15
Send PM
Re: If |x–1|<4, so: [#permalink]
2
Kudos
I can't understand why x>-4 is the correct answer.

Since we have -3<x<5 , -3 is out of the range and in x> -4 -3 is included in the number line.

Besides that, X>-4 we can have x=10, as an exemple, wich is not true since x<5.

Can someone help me with this points?

tks
Intern
Intern
Joined: 07 Jul 2017
Posts: 7
Own Kudos [?]: 3 [0]
Given Kudos: 14
Send PM
Re: If |x–1|<4, so: [#permalink]
Bunuel wrote:
vitorpteixeira wrote:
If |x–1|<4, so:

(A) x > 3
(B) x ≤ 4
(C) -4< x < 4
(D) x > -4
(E) none of the above


\(|x–1|<4\);

\(-4<x-1<4\);

\(-3<x<5\).

Any x from that range will be more than -4.

Answer: D.

P.S. What is the source of this question?


Hi Bunuel, thanks for the explanation. I tend to approach this questions in the wrong way, trying to find an answer choice that matches with the range found through the inequalities. For instance, in this case I was looking for -3<x<5 (or something more restrictive, such as -2 < x < -5), which led me to pick answer E.

Given your explanation, I understand that I should look for that condition that holds for every x in the range...but I still have a huge doubt.
You say it is (D) x > -4 because any X from that range will be more than -4. Following your logic, wouldn't also B) x ≤ 4 be true? Our range is -3<x<5, which means that every x will be less or equal to 4.

Is there something I am not getting right?

Your help is super appreciated
Math Expert
Joined: 02 Sep 2009
Posts: 93473
Own Kudos [?]: 628902 [2]
Given Kudos: 82085
Send PM
Re: If |x–1|<4, so: [#permalink]
2
Kudos
Expert Reply
delid wrote:
Bunuel wrote:
vitorpteixeira wrote:
If |x–1|<4, so:

(A) x > 3
(B) x ≤ 4
(C) -4< x < 4
(D) x > -4
(E) none of the above


\(|x–1|<4\);

\(-4<x-1<4\);

\(-3<x<5\).

Any x from that range will be more than -4.

Answer: D.

P.S. What is the source of this question?


Hi Bunuel, thanks for the explanation. I tend to approach this questions in the wrong way, trying to find an answer choice that matches with the range found through the inequalities. For instance, in this case I was looking for -3<x<5 (or something more restrictive, such as -2 < x < -5), which led me to pick answer E.

Given your explanation, I understand that I should look for that condition that holds for every x in the range...but I still have a huge doubt.
You say it is (D) x > -4 because any X from that range will be more than -4. Following your logic, wouldn't also B) x ≤ 4 be true? Our range is -3<x<5, which means that every x will be less or equal to 4.

Is there something I am not getting right?

Your help is super appreciated


For \(-3<x<5\), (B) x ≤ 4 won't always be true. For example, x could be 4.5 (4.5 IS between -3 and 5) and in this case B is not true.
Intern
Intern
Joined: 13 Oct 2017
Posts: 1
Own Kudos [?]: 0 [0]
Given Kudos: 0
Send PM
Re: If |x–1|<4, so: [#permalink]
|x–1|<4


−3<x<5

Any x from that range will be more than -4.

Answer: D.
Math Expert
Joined: 02 Sep 2009
Posts: 93473
Own Kudos [?]: 628902 [0]
Given Kudos: 82085
Send PM
Re: If |x–1|<4, so: [#permalink]
Expert Reply
vitorpteixeira wrote:
If |x–1|<4, so:

(A) x > 3
(B) x ≤ 4
(C) -4< x < 4
(D) x > -4
(E) none of the above


Check other similar questions from Trickiest Inequality Questions Type: Confusing Ranges (part of our Special Questions Directory).
Current Student
Joined: 04 Sep 2017
Status:Booth 1Y
Posts: 278
Own Kudos [?]: 1167 [1]
Given Kudos: 228
Location: United States (IL)
Concentration: Technology, Leadership
GMAT 1: 690 Q44 V41
GMAT 2: 730 Q50 V38
GPA: 3.62
WE:Sales (Computer Software)
Send PM
Re: If |x–1|<4, so: [#permalink]
1
Kudos
Wouldn't this question be better off as a "which of the following must be true" question? Does the actual test do stuff like this?
Intern
Intern
Joined: 10 Jun 2019
Posts: 7
Own Kudos [?]: 3 [0]
Given Kudos: 7
Send PM
If |x–1|<4, so: [#permalink]
On solving the inequality, we get x<5 and x>-3.
Among the options, we have x>-4, which is already satisfied given that x>-3 and since -4<-3, we can conclude that x>-4.
Hence, Option D is the right choice.
GMAT Club Legend
GMAT Club Legend
Joined: 03 Oct 2013
Affiliations: CrackVerbal
Posts: 4940
Own Kudos [?]: 7707 [1]
Given Kudos: 216
Location: India
Send PM
Re: If |x–1|<4, so: [#permalink]
1
Kudos
delid wrote:
Bunuel wrote:
vitorpteixeira wrote:
If |x–1|<4, so:

(A) x > 3
(B) x ≤ 4
(C) -4< x < 4
(D) x > -4
(E) none of the above


\(|x–1|<4\);

\(-4<x-1<4\);

\(-3<x<5\).

Any x from that range will be more than -4.

Answer: D.

P.S. What is the source of this question?


Hi Bunuel, thanks for the explanation. I tend to approach this questions in the wrong way, trying to find an answer choice that matches with the range found through the inequalities. For instance, in this case I was looking for -3<x<5 (or something more restrictive, such as -2 < x < -5), which led me to pick answer E.

Given your explanation, I understand that I should look for that condition that holds for every x in the range...but I still have a huge doubt.
You say it is (D) x > -4 because any X from that range will be more than -4. Following your logic, wouldn't also B) x ≤ 4 be true? Our range is -3<x<5, which means that every x will be less or equal to 4.

Is there something I am not getting right?

Your help is super appreciated



Although this is a relatively simple question on absolute values (modulus), the way in which the options have been framed can put you off.

The reason for this is that, you go in to the solution, expecting to see a range that you think you’ll get when you solve the given inequality. But, in reality, none of the options represent the exact range that you will get when you solve the inequality.
Therefore, it becomes important to understand that , here, we are trying to find out a range of values, which in turn fully covers the range which satisfies the given inequality.

If |x-a| < y, the range of x that satisfies the inequality will be a-y < x < a+y. When we apply this to the given inequality, |x-1| < 4, we can say that -3<x<5. Let us represent this on a number line, as shown below:

Attachment:
21st June 2019 - Reply 3 - 1.JPG
21st June 2019 - Reply 3 - 1.JPG [ 14.93 KiB | Viewed 9547 times ]


Let us evaluate the options now.

Option 1 says x>3. This means that x can be any value from 3 to infinity. Clearly, only a portion of this range satisfies our inequality i.e. 3<x<5.
So, quite obviously, you cannot say that all values which satisfy |x-1|<4 fall in the range of x>3.

Attachment:
21st June 2019 - Reply 3 - 2.JPG
21st June 2019 - Reply 3 - 2.JPG [ 16.29 KiB | Viewed 9526 times ]


A similar thing works against option B.

With option C, although some numbers are within the range, some numbers aren’t. Like for example, x = 4.5 satisfies the given inequality but is not within -4<x<4.

When it comes to option D, we can say that all the values that satisfy our inequality are greater than -4. This means, when I pick any value from the range satisfying the inequality, it will always qualify as true when compared with option D.

The same cannot be said about the other options, as we have demonstrated.
So, the correct answer is D.

The confusion that this question has created could have been nullified if the question was framed as a ‘Must be’ question. For example “ If |x-1|<4, which of the following must be true for the values of x that satisfy the inequality?” . Essentially, this is what the question is testing you on.

Hope this helps!
Intern
Intern
Joined: 21 Aug 2020
Posts: 10
Own Kudos [?]: 0 [0]
Given Kudos: 12
Send PM
Re: If |x1|<4, so: [#permalink]
Bunuel wrote:
vitorpteixeira wrote:
If |x–1|<4, so:

(A) x > 3
(B) x ≤ 4
(C) -4< x < 4
(D) x > -4
(E) none of the above


\(|x–1|<4\);

\(-4<x-1<4\);

\(-3<x<5\).

Any x from that range will be more than -4.

Answer: D.

P.S. What is the source of this question?


I am in a huge confusion. The range is -3<x<5 and I eliminated all the choices based on this range including D. Because x>-4 does not give an upper limit and it can be 10 or smth more. What I miss here?
RC & DI Moderator
Joined: 02 Aug 2009
Status:Math and DI Expert
Posts: 11370
Own Kudos [?]: 33133 [1]
Given Kudos: 314
Send PM
Re: If |x1|<4, so: [#permalink]
1
Kudos
Expert Reply
Sevil92 wrote:
Bunuel wrote:
vitorpteixeira wrote:
If |x–1|<4, so:

(A) x > 3
(B) x ≤ 4
(C) -4< x < 4
(D) x > -4
(E) none of the above


\(|x–1|<4\);

\(-4<x-1<4\);

\(-3<x<5\).

Any x from that range will be more than -4.

Answer: D.

P.S. What is the source of this question?


I am in a huge confusion. The range is -3<x<5 and I eliminated all the choices based on this range including D. Because x>-4 does not give an upper limit and it can be 10 or smth more. What I miss here?


Hi

A simpler and similar question would be

If x>2, then
‘x is positive’ is correct since x has to be positive. x>0 may contain extra values, those between 0 to 2, but it also encompasses the entire possible range.
Similar is the case here. We are not looking at exact range here, but at what MUST be true.
User avatar
Intern
Intern
Joined: 08 May 2023
Posts: 1
Own Kudos [?]: 0 [0]
Given Kudos: 1
Send PM
Re: If |x1|<4, so: [#permalink]
Bunuel wrote:
vitorpteixeira wrote:
If |x–1|<4, so:

(A) x > 3
(B) x ≤ 4
(C) -4< x < 4
(D) x > -4
(E) none of the above


\(|x–1|<4\);

\(-4<x-1<4\);

\(-3<x<5\).

Any x from that range will be more than -4.

Answer: D.

P.S. What is the source of this question?



How? I dont get it.

When we write -3<x how can this be x>-4 🤷‍♂️. Isn't it mean x can be -3?

Posted from my mobile device
GMAT Club Bot
Re: If |x1|<4, so: [#permalink]